For what $n$ is $U_n$ cyclic?












23















When can we say a multiplicative group of integers modulo $n$, i.e., $U_n$ is cyclic?




$$U_n={a inmathbb Z_n mid gcd(a,n)=1 }$$



I searched the internet but did not get a clear idea.










share|cite|improve this question





























    23















    When can we say a multiplicative group of integers modulo $n$, i.e., $U_n$ is cyclic?




    $$U_n={a inmathbb Z_n mid gcd(a,n)=1 }$$



    I searched the internet but did not get a clear idea.










    share|cite|improve this question



























      23












      23








      23


      17






      When can we say a multiplicative group of integers modulo $n$, i.e., $U_n$ is cyclic?




      $$U_n={a inmathbb Z_n mid gcd(a,n)=1 }$$



      I searched the internet but did not get a clear idea.










      share|cite|improve this question
















      When can we say a multiplicative group of integers modulo $n$, i.e., $U_n$ is cyclic?




      $$U_n={a inmathbb Z_n mid gcd(a,n)=1 }$$



      I searched the internet but did not get a clear idea.







      abstract-algebra group-theory cyclic-groups






      share|cite|improve this question















      share|cite|improve this question













      share|cite|improve this question




      share|cite|improve this question








      edited Feb 7 '16 at 11:17









      Rudy the Reindeer

      26.4k1690239




      26.4k1690239










      asked Feb 26 '13 at 12:58









      SankhaSankha

      575622




      575622






















          3 Answers
          3






          active

          oldest

          votes


















          24














          So $U_n$ is the group of units in $mathbb{Z}/nmathbb{Z}$.



          Write the prime decomposition
          $$
          n=p_1^{alpha_1}cdots p_r^{alpha_r}.
          $$



          By the Chinese remainder theorem
          $$
          mathbb{Z}/nmathbb{Z}=mathbb{Z}/p_1^{alpha_1}mathbb{Z}timesldotstimesmathbb{Z}/p_r^{alpha_r}mathbb{Z}
          $$
          so
          $$
          U_n=U_{p_1^{alpha_1}}timesldotstimes U_{p_r^{alpha_r}}.
          $$



          For powers of $2$, we have
          $$
          U_2={0}
          $$
          and for $kgeq 2$
          $$
          U_{2^k}=mathbb{Z}/2mathbb{Z}times mathbb{Z}/2^{k-2}mathbb{Z}.
          $$



          For odd primes $p$,
          $$
          U_{p^alpha}=mathbb{Z}/phi(p^alpha)mathbb{Z}=mathbb{Z}/p^{alpha-1}(p-1)mathbb{Z}.
          $$



          So you see now that $U_n$ is cyclic if and only if
          $$
          n=2,4,p^alpha,2p^{alpha}
          $$
          where $p$ is an odd prime.



          Here is a reference.






          share|cite|improve this answer



















          • 1




            Why is it true that $U_{p^k}=mathbb{Z}/2mathbb{Z}timesmathbb{Z}/2^{k-2}mathbb{Z}$?
            – Rasputin
            Jan 20 '17 at 20:03










          • Julien, why doesn't the even prime work please?
            – BCLC
            Oct 17 '18 at 11:48



















          9














          $U_n$ is cyclic iff $n$ is $2$, $4$, $p^k$, or $2p^k$, where $p$ is an odd prime.



          The proof follows from the Chinese Remainder Theorem for rings and the fact that $C_m times C_n$ is cyclic iff $(m,n)=1$ (here $C_n$ is the cyclic group of order $n$).



          The hard part is proving that $U_p$ is cyclic and this follows from the fact that $mathbb Z/p$ is a field and that $n = sum_{dmid n} phi(d)$.



          Any book on elementary number theory has a proof of this theorem. See for instance André Weil's Number theory for beginners, Leveque's Fundamentals of Number Theory, and Bolker's Elementary Number Theory.






          share|cite|improve this answer























          • @julien, thanks, fixed.
            – lhf
            Feb 26 '13 at 13:26



















          5














          Here "cyclic if and only if $varphi(n)=lambda(n)$" but there's no proof - the proof is elementary but very tricky.






          share|cite|improve this answer























            Your Answer





            StackExchange.ifUsing("editor", function () {
            return StackExchange.using("mathjaxEditing", function () {
            StackExchange.MarkdownEditor.creationCallbacks.add(function (editor, postfix) {
            StackExchange.mathjaxEditing.prepareWmdForMathJax(editor, postfix, [["$", "$"], ["\\(","\\)"]]);
            });
            });
            }, "mathjax-editing");

            StackExchange.ready(function() {
            var channelOptions = {
            tags: "".split(" "),
            id: "69"
            };
            initTagRenderer("".split(" "), "".split(" "), channelOptions);

            StackExchange.using("externalEditor", function() {
            // Have to fire editor after snippets, if snippets enabled
            if (StackExchange.settings.snippets.snippetsEnabled) {
            StackExchange.using("snippets", function() {
            createEditor();
            });
            }
            else {
            createEditor();
            }
            });

            function createEditor() {
            StackExchange.prepareEditor({
            heartbeatType: 'answer',
            autoActivateHeartbeat: false,
            convertImagesToLinks: true,
            noModals: true,
            showLowRepImageUploadWarning: true,
            reputationToPostImages: 10,
            bindNavPrevention: true,
            postfix: "",
            imageUploader: {
            brandingHtml: "Powered by u003ca class="icon-imgur-white" href="https://imgur.com/"u003eu003c/au003e",
            contentPolicyHtml: "User contributions licensed under u003ca href="https://creativecommons.org/licenses/by-sa/3.0/"u003ecc by-sa 3.0 with attribution requiredu003c/au003e u003ca href="https://stackoverflow.com/legal/content-policy"u003e(content policy)u003c/au003e",
            allowUrls: true
            },
            noCode: true, onDemand: true,
            discardSelector: ".discard-answer"
            ,immediatelyShowMarkdownHelp:true
            });


            }
            });














            draft saved

            draft discarded


















            StackExchange.ready(
            function () {
            StackExchange.openid.initPostLogin('.new-post-login', 'https%3a%2f%2fmath.stackexchange.com%2fquestions%2f314846%2ffor-what-n-is-u-n-cyclic%23new-answer', 'question_page');
            }
            );

            Post as a guest















            Required, but never shown

























            3 Answers
            3






            active

            oldest

            votes








            3 Answers
            3






            active

            oldest

            votes









            active

            oldest

            votes






            active

            oldest

            votes









            24














            So $U_n$ is the group of units in $mathbb{Z}/nmathbb{Z}$.



            Write the prime decomposition
            $$
            n=p_1^{alpha_1}cdots p_r^{alpha_r}.
            $$



            By the Chinese remainder theorem
            $$
            mathbb{Z}/nmathbb{Z}=mathbb{Z}/p_1^{alpha_1}mathbb{Z}timesldotstimesmathbb{Z}/p_r^{alpha_r}mathbb{Z}
            $$
            so
            $$
            U_n=U_{p_1^{alpha_1}}timesldotstimes U_{p_r^{alpha_r}}.
            $$



            For powers of $2$, we have
            $$
            U_2={0}
            $$
            and for $kgeq 2$
            $$
            U_{2^k}=mathbb{Z}/2mathbb{Z}times mathbb{Z}/2^{k-2}mathbb{Z}.
            $$



            For odd primes $p$,
            $$
            U_{p^alpha}=mathbb{Z}/phi(p^alpha)mathbb{Z}=mathbb{Z}/p^{alpha-1}(p-1)mathbb{Z}.
            $$



            So you see now that $U_n$ is cyclic if and only if
            $$
            n=2,4,p^alpha,2p^{alpha}
            $$
            where $p$ is an odd prime.



            Here is a reference.






            share|cite|improve this answer



















            • 1




              Why is it true that $U_{p^k}=mathbb{Z}/2mathbb{Z}timesmathbb{Z}/2^{k-2}mathbb{Z}$?
              – Rasputin
              Jan 20 '17 at 20:03










            • Julien, why doesn't the even prime work please?
              – BCLC
              Oct 17 '18 at 11:48
















            24














            So $U_n$ is the group of units in $mathbb{Z}/nmathbb{Z}$.



            Write the prime decomposition
            $$
            n=p_1^{alpha_1}cdots p_r^{alpha_r}.
            $$



            By the Chinese remainder theorem
            $$
            mathbb{Z}/nmathbb{Z}=mathbb{Z}/p_1^{alpha_1}mathbb{Z}timesldotstimesmathbb{Z}/p_r^{alpha_r}mathbb{Z}
            $$
            so
            $$
            U_n=U_{p_1^{alpha_1}}timesldotstimes U_{p_r^{alpha_r}}.
            $$



            For powers of $2$, we have
            $$
            U_2={0}
            $$
            and for $kgeq 2$
            $$
            U_{2^k}=mathbb{Z}/2mathbb{Z}times mathbb{Z}/2^{k-2}mathbb{Z}.
            $$



            For odd primes $p$,
            $$
            U_{p^alpha}=mathbb{Z}/phi(p^alpha)mathbb{Z}=mathbb{Z}/p^{alpha-1}(p-1)mathbb{Z}.
            $$



            So you see now that $U_n$ is cyclic if and only if
            $$
            n=2,4,p^alpha,2p^{alpha}
            $$
            where $p$ is an odd prime.



            Here is a reference.






            share|cite|improve this answer



















            • 1




              Why is it true that $U_{p^k}=mathbb{Z}/2mathbb{Z}timesmathbb{Z}/2^{k-2}mathbb{Z}$?
              – Rasputin
              Jan 20 '17 at 20:03










            • Julien, why doesn't the even prime work please?
              – BCLC
              Oct 17 '18 at 11:48














            24












            24








            24






            So $U_n$ is the group of units in $mathbb{Z}/nmathbb{Z}$.



            Write the prime decomposition
            $$
            n=p_1^{alpha_1}cdots p_r^{alpha_r}.
            $$



            By the Chinese remainder theorem
            $$
            mathbb{Z}/nmathbb{Z}=mathbb{Z}/p_1^{alpha_1}mathbb{Z}timesldotstimesmathbb{Z}/p_r^{alpha_r}mathbb{Z}
            $$
            so
            $$
            U_n=U_{p_1^{alpha_1}}timesldotstimes U_{p_r^{alpha_r}}.
            $$



            For powers of $2$, we have
            $$
            U_2={0}
            $$
            and for $kgeq 2$
            $$
            U_{2^k}=mathbb{Z}/2mathbb{Z}times mathbb{Z}/2^{k-2}mathbb{Z}.
            $$



            For odd primes $p$,
            $$
            U_{p^alpha}=mathbb{Z}/phi(p^alpha)mathbb{Z}=mathbb{Z}/p^{alpha-1}(p-1)mathbb{Z}.
            $$



            So you see now that $U_n$ is cyclic if and only if
            $$
            n=2,4,p^alpha,2p^{alpha}
            $$
            where $p$ is an odd prime.



            Here is a reference.






            share|cite|improve this answer














            So $U_n$ is the group of units in $mathbb{Z}/nmathbb{Z}$.



            Write the prime decomposition
            $$
            n=p_1^{alpha_1}cdots p_r^{alpha_r}.
            $$



            By the Chinese remainder theorem
            $$
            mathbb{Z}/nmathbb{Z}=mathbb{Z}/p_1^{alpha_1}mathbb{Z}timesldotstimesmathbb{Z}/p_r^{alpha_r}mathbb{Z}
            $$
            so
            $$
            U_n=U_{p_1^{alpha_1}}timesldotstimes U_{p_r^{alpha_r}}.
            $$



            For powers of $2$, we have
            $$
            U_2={0}
            $$
            and for $kgeq 2$
            $$
            U_{2^k}=mathbb{Z}/2mathbb{Z}times mathbb{Z}/2^{k-2}mathbb{Z}.
            $$



            For odd primes $p$,
            $$
            U_{p^alpha}=mathbb{Z}/phi(p^alpha)mathbb{Z}=mathbb{Z}/p^{alpha-1}(p-1)mathbb{Z}.
            $$



            So you see now that $U_n$ is cyclic if and only if
            $$
            n=2,4,p^alpha,2p^{alpha}
            $$
            where $p$ is an odd prime.



            Here is a reference.







            share|cite|improve this answer














            share|cite|improve this answer



            share|cite|improve this answer








            edited Mar 18 '16 at 21:25









            user26857

            39.3k124083




            39.3k124083










            answered Feb 26 '13 at 13:24









            JulienJulien

            38.5k358129




            38.5k358129








            • 1




              Why is it true that $U_{p^k}=mathbb{Z}/2mathbb{Z}timesmathbb{Z}/2^{k-2}mathbb{Z}$?
              – Rasputin
              Jan 20 '17 at 20:03










            • Julien, why doesn't the even prime work please?
              – BCLC
              Oct 17 '18 at 11:48














            • 1




              Why is it true that $U_{p^k}=mathbb{Z}/2mathbb{Z}timesmathbb{Z}/2^{k-2}mathbb{Z}$?
              – Rasputin
              Jan 20 '17 at 20:03










            • Julien, why doesn't the even prime work please?
              – BCLC
              Oct 17 '18 at 11:48








            1




            1




            Why is it true that $U_{p^k}=mathbb{Z}/2mathbb{Z}timesmathbb{Z}/2^{k-2}mathbb{Z}$?
            – Rasputin
            Jan 20 '17 at 20:03




            Why is it true that $U_{p^k}=mathbb{Z}/2mathbb{Z}timesmathbb{Z}/2^{k-2}mathbb{Z}$?
            – Rasputin
            Jan 20 '17 at 20:03












            Julien, why doesn't the even prime work please?
            – BCLC
            Oct 17 '18 at 11:48




            Julien, why doesn't the even prime work please?
            – BCLC
            Oct 17 '18 at 11:48











            9














            $U_n$ is cyclic iff $n$ is $2$, $4$, $p^k$, or $2p^k$, where $p$ is an odd prime.



            The proof follows from the Chinese Remainder Theorem for rings and the fact that $C_m times C_n$ is cyclic iff $(m,n)=1$ (here $C_n$ is the cyclic group of order $n$).



            The hard part is proving that $U_p$ is cyclic and this follows from the fact that $mathbb Z/p$ is a field and that $n = sum_{dmid n} phi(d)$.



            Any book on elementary number theory has a proof of this theorem. See for instance André Weil's Number theory for beginners, Leveque's Fundamentals of Number Theory, and Bolker's Elementary Number Theory.






            share|cite|improve this answer























            • @julien, thanks, fixed.
              – lhf
              Feb 26 '13 at 13:26
















            9














            $U_n$ is cyclic iff $n$ is $2$, $4$, $p^k$, or $2p^k$, where $p$ is an odd prime.



            The proof follows from the Chinese Remainder Theorem for rings and the fact that $C_m times C_n$ is cyclic iff $(m,n)=1$ (here $C_n$ is the cyclic group of order $n$).



            The hard part is proving that $U_p$ is cyclic and this follows from the fact that $mathbb Z/p$ is a field and that $n = sum_{dmid n} phi(d)$.



            Any book on elementary number theory has a proof of this theorem. See for instance André Weil's Number theory for beginners, Leveque's Fundamentals of Number Theory, and Bolker's Elementary Number Theory.






            share|cite|improve this answer























            • @julien, thanks, fixed.
              – lhf
              Feb 26 '13 at 13:26














            9












            9








            9






            $U_n$ is cyclic iff $n$ is $2$, $4$, $p^k$, or $2p^k$, where $p$ is an odd prime.



            The proof follows from the Chinese Remainder Theorem for rings and the fact that $C_m times C_n$ is cyclic iff $(m,n)=1$ (here $C_n$ is the cyclic group of order $n$).



            The hard part is proving that $U_p$ is cyclic and this follows from the fact that $mathbb Z/p$ is a field and that $n = sum_{dmid n} phi(d)$.



            Any book on elementary number theory has a proof of this theorem. See for instance André Weil's Number theory for beginners, Leveque's Fundamentals of Number Theory, and Bolker's Elementary Number Theory.






            share|cite|improve this answer














            $U_n$ is cyclic iff $n$ is $2$, $4$, $p^k$, or $2p^k$, where $p$ is an odd prime.



            The proof follows from the Chinese Remainder Theorem for rings and the fact that $C_m times C_n$ is cyclic iff $(m,n)=1$ (here $C_n$ is the cyclic group of order $n$).



            The hard part is proving that $U_p$ is cyclic and this follows from the fact that $mathbb Z/p$ is a field and that $n = sum_{dmid n} phi(d)$.



            Any book on elementary number theory has a proof of this theorem. See for instance André Weil's Number theory for beginners, Leveque's Fundamentals of Number Theory, and Bolker's Elementary Number Theory.







            share|cite|improve this answer














            share|cite|improve this answer



            share|cite|improve this answer








            edited Feb 26 '13 at 14:59









            Michael Hardy

            1




            1










            answered Feb 26 '13 at 13:11









            lhflhf

            163k10168388




            163k10168388












            • @julien, thanks, fixed.
              – lhf
              Feb 26 '13 at 13:26


















            • @julien, thanks, fixed.
              – lhf
              Feb 26 '13 at 13:26
















            @julien, thanks, fixed.
            – lhf
            Feb 26 '13 at 13:26




            @julien, thanks, fixed.
            – lhf
            Feb 26 '13 at 13:26











            5














            Here "cyclic if and only if $varphi(n)=lambda(n)$" but there's no proof - the proof is elementary but very tricky.






            share|cite|improve this answer




























              5














              Here "cyclic if and only if $varphi(n)=lambda(n)$" but there's no proof - the proof is elementary but very tricky.






              share|cite|improve this answer


























                5












                5








                5






                Here "cyclic if and only if $varphi(n)=lambda(n)$" but there's no proof - the proof is elementary but very tricky.






                share|cite|improve this answer














                Here "cyclic if and only if $varphi(n)=lambda(n)$" but there's no proof - the proof is elementary but very tricky.







                share|cite|improve this answer














                share|cite|improve this answer



                share|cite|improve this answer








                edited Dec 16 '13 at 9:18

























                answered Feb 26 '13 at 13:07







                user58512





































                    draft saved

                    draft discarded




















































                    Thanks for contributing an answer to Mathematics Stack Exchange!


                    • Please be sure to answer the question. Provide details and share your research!

                    But avoid



                    • Asking for help, clarification, or responding to other answers.

                    • Making statements based on opinion; back them up with references or personal experience.


                    Use MathJax to format equations. MathJax reference.


                    To learn more, see our tips on writing great answers.




                    draft saved


                    draft discarded














                    StackExchange.ready(
                    function () {
                    StackExchange.openid.initPostLogin('.new-post-login', 'https%3a%2f%2fmath.stackexchange.com%2fquestions%2f314846%2ffor-what-n-is-u-n-cyclic%23new-answer', 'question_page');
                    }
                    );

                    Post as a guest















                    Required, but never shown





















































                    Required, but never shown














                    Required, but never shown












                    Required, but never shown







                    Required, but never shown

































                    Required, but never shown














                    Required, but never shown












                    Required, but never shown







                    Required, but never shown







                    Popular posts from this blog

                    Mario Kart Wii

                    What does “Dominus providebit” mean?

                    Antonio Litta Visconti Arese